Sequential Compactness in Metric Spaces: Proving Compactness of Product Metric

  • Thread starter Thread starter Ted123
  • Start date Start date
Click For Summary
The discussion focuses on proving that the product of two sequentially compact metric spaces, (X, d_X) and (Y, d_Y), is also sequentially compact when equipped with the product metric d_{X×Y}. The user demonstrates that any sequence in the product space (X×Y) has convergent subsequences in both X and Y due to their sequential compactness. They conclude that the subsequence in the product space converges to a point in X×Y, confirming the sequential compactness of the product space. The user verifies that the convergence holds in the product metric by showing that the distance approaches zero. The proof is deemed correct and complete.
Ted123
Messages
428
Reaction score
0

Homework Statement



Suppose (X,d_X) and (Y,d_Y) are sequentially compact metric spaces. Show that (X\times Y, d_{X\times Y}) is sequentially compact where d_{X\times Y} ((x_1,y_1),(x_2,y_2)) = d_X(x_1,x_2) + d_Y(y_1,y_2) is the product metric.

The Attempt at a Solution



Suppose (x_n)_{n\in\mathbb{N}} is a sequence in X and (y_n)_{n\in\mathbb{N}} is a sequence in Y.

Then since X,Y are sequentially compact (x_n)_{n\in\mathbb{N}} and (y_n)_{n\in\mathbb{N}} have convergent subsequences, say x_{n_k} \to x\in X and y_{n_k} \to y\in Y as k\to\infty.

It follows that if (x_n,y_n)_{n\in\mathbb{N}} is a sequence in X\times Y with subsequence (x_{n_k},y_{n_k})_{k\in\mathbb{N}} then (x_{n_k},y_{n_k}) \to (x,y)\in X\times Y as k\to\infty.

Is this OK so far? Do I now need to show that (x_{n_k},y_{n_k}) \to (x,y) in the metric d_{X\times Y} ?

In which case:

d_{X\times Y}((x_{n_k} , y_{n_k}),(x,y)) = d_X(x_{n_k} , x) + d_Y(y_{n_k},y) \to 0+0=0

so (x_{n_k},y_{n_k}) \to (x,y) in the metric d_{X\times Y}.
 
Last edited:
Physics news on Phys.org
Looks ok!
 
Question: A clock's minute hand has length 4 and its hour hand has length 3. What is the distance between the tips at the moment when it is increasing most rapidly?(Putnam Exam Question) Answer: Making assumption that both the hands moves at constant angular velocities, the answer is ## \sqrt{7} .## But don't you think this assumption is somewhat doubtful and wrong?

Similar threads

Replies
2
Views
2K
  • · Replies 1 ·
Replies
1
Views
2K
  • · Replies 18 ·
Replies
18
Views
3K
  • · Replies 12 ·
Replies
12
Views
2K
  • · Replies 3 ·
Replies
3
Views
2K
  • · Replies 13 ·
Replies
13
Views
3K
  • · Replies 14 ·
Replies
14
Views
2K
  • · Replies 6 ·
Replies
6
Views
2K
  • · Replies 10 ·
Replies
10
Views
3K
  • · Replies 5 ·
Replies
5
Views
2K